6

積分botを解けるだけ解く, その1

364
1
$$$$

この記事では, 積分botの積分を僕が証明できるだけ解き続けたいと思います. とはいえ, 全部厳密な証明をしているといつまでたっても終わらない気がするので, かなり略すところがあると思います. とりあえず, ある程度前のツイートから順番にいきたいと思います.

1個目( https://twitter.com/integralsbot/status/1361111343697453060 )

$$\begin{eqnarray*} \int_0^{\pi}\sin^{\mu-1}x\sin bx\,dx=\frac{2^{1-\mu}\pi\Gamma(\mu)\sin\frac{\pi b}{2}}{\Gamma\left(\frac{1+\mu-b}{2}\right)\Gamma\left(\frac{1+\mu+b}{2}\right)} \end{eqnarray*}$$

これはベータ関数のもう一つの形というやつですね. さっそく証明を書いていきます.

積分路1 積分路1

上のような積分路$C$を考えます.

$$\begin{eqnarray*} \int_Cx^{a-1}(1-x)^{b-1}\,dx&=&(1-e^{2\pi ia})\int_0^1x^{a-1}(1-x)^{b-1}\,dx+i\int_0^{2\pi}e^{iax}(1-e^{ix})^{b-1}\,dx\\ &=&-2ie^{i\pi a}B(a,b)\sin\pi a-e^{-i\pi b/2}\int_0^{2\pi}e^{i(a+(b-1)/2)x}\left(2\sin\frac x2\right)^{b-1}\,dx \end{eqnarray*}$$

一方, $\ln$の主値は上の曲線内で$1$価正則に取れるので, この積分は$0$になります. よって,

$$\begin{eqnarray*} \int_0^{2\pi}e^{i(a+(b-1)/2)x}\left(2\sin\frac x2\right)^{b-1}\,dx&=&2e^{i\pi (a+(b-1)/2)}B(a,b)\sin\pi a\\ &=&\frac{2\pi\Gamma(b)e^{i\pi(a+(b-1)/2)}}{\Gamma(1-a)\Gamma(a+b)} \end{eqnarray*}$$

$a\mapsto \frac{1-b+c}2$と置き換えて,
$$\begin{eqnarray*} \int_0^{2\pi}e^{icx/2}\left(2\sin\frac x2\right)^{b-1}\,dx&=&\frac{2\pi\Gamma(b)e^{i\pi c/2}}{\Gamma\left(\frac{1-c+b}{2}\right)\Gamma\left(\frac{1+c+b}2\right)}\\ \int_0^{\pi}e^{icx}\sin^{b-1} x\,dx&=&\frac{2^{1-b}\pi\Gamma(b)e^{i\pi c/2}}{\Gamma\left(\frac{1-c+b}{2}\right)\Gamma\left(\frac{1+c+b}2\right)} \end{eqnarray*}$$
この虚部を考えることにより, 証明ができました.

2個目( https://twitter.com/integralsbot/status/1361113203866886145 )

$$\begin{eqnarray} \int_0^1\ln\frac{1+ax}{1-ax}\frac{dx}{x\sqrt{1-x^2}}=\pi\arcsin a \end{eqnarray}$$

もとのツイートでは$\sin^{-1}$になっていますが, 僕は$\arcsin$派なんですよね. 証明を書いていきます.

$$\begin{eqnarray*} \ln\frac{1+ax}{1-ax}=2\sum_{0\leq n}\frac{(ax)^{2n+1}}{2n+1} \end{eqnarray*}$$
なので, 項別に積分していくと,
$$\begin{eqnarray*} \int_0^1\ln\frac{1+ax}{1-ax}\frac{dx}{x\sqrt{1-x^2}}&=&\sum_{0\leq n} \frac{2}{2n+1}a^{2n+1}\int_0^1x^{2n}(1-x^2)^{-1/2}\,dx\\ &=&\sum_{0\leq n}\frac{a^{2n+1}}{2n+1}\int_0^1x^{n-1/2}(1-x)^{-1/2}\,dx\\ &=&\sum_{0\leq n}\frac{a^{2n+1}}{2n+1}\frac{\Gamma\left(\frac 12\right)\Gamma\left(\frac 12+n\right)}{\Gamma(n+1)}\\ &=&\pi\sum_{0\leq n}\frac{\left(\frac 12\right)_n}{n!(2n+1)}a^{2n+1}\\ &=&\pi\arcsin a \end{eqnarray*}$$
となって証明ができました.

3個目( https://twitter.com/integralsbot/status/1361120186884120576 )

$$\begin{eqnarray*} \int_{-\infty}^{\infty}\ln(2-2\cos x^2)\,dx=-\sqrt{2\pi}\zeta\left(\frac 32\right) \end{eqnarray*}$$

こういうゼータ関数が出てくるのは, Mellin変換っぽいですね.

$$\begin{eqnarray*} \int_{-\infty}^{\infty}\ln(2-2\cos x^2)\,dx&=&2\int_0^{\infty}\ln(2-2\cos x^2)\,dx\\ &=&4\int_0^{\infty}\ln\left|2\sin\frac {x^2}2\right|\,dx\\ &=&2\int_0^{\infty}x^{-1/2}\ln\left|2\sin\frac x2\right|\,dx \end{eqnarray*}$$
$0\lt \Re s$として,
$$\begin{eqnarray*} \int_0^{\infty}x^{s-1}\ln(1-e^{-ix})\,dx&=&-\sum_{0\lt n}\frac 1n\int_0^{\infty}x^{s-1}e^{-inx}\,dx\\ &=&-i^{-s}\Gamma(s)\sum_{0\lt n}\frac 1{n^{s+1}}\\ &=&-i^{-s}\Gamma(s)\zeta(s+1) \end{eqnarray*}$$
$s=\frac 12$として, これの実部をとることにより,
$$\begin{eqnarray*} 2\int_0^{\infty}x^{-1/2}\ln\left|2\sin\frac x2\right|\,dx=-\sqrt{2\pi}\zeta\left(\frac 32\right) \end{eqnarray*}$$
を得ます.

4個目( https://twitter.com/integralsbot/status/1361120737830965249 )

$$\begin{eqnarray*} \int_0^{\infty}\frac{\cos x^2-\cos x}{x}\,dx=\frac{\gamma}2 \end{eqnarray*}$$

とりあえず, Mellin変換しよう.

$$\begin{eqnarray*} \int_0^{\infty}x^{s-1}(\cos x^2-\cos x)\,dx&=&\frac 12\Gamma\left(\frac s2\right)\cos\frac{\pi s}{4}-\Gamma(s)\cos\frac{\pi s}{2}\\ &=&\frac 12\left(\frac 2s-\gamma+O(s)\right)(1+O(s^2))-\left(\frac 1s-\gamma+O(s)\right)(1+O(s^2))\\ &=&\frac{\gamma}{2}+O(s) \end{eqnarray*}$$
ここで, $s\to +0$とすればよい.

5個目( https://twitter.com/integralsbot/status/1361137724728737797 )

$$\begin{eqnarray*} \int_0^{\pi/2}\frac 1{\tan x}\ln\left(\frac{1+(\tan x+\sinh y)^2\cosh^2y}{1+(\tan x-\sinh y)^2\cosh^2 y}\right)\,dx=2\pi\arctan\tanh\frac x2 \end{eqnarray*}$$

ん?これ右辺に$x$出てきてるけど多分$y$だよね, と思って解いていきます.

まず, 少し準備をします.
$$\begin{eqnarray*} \int_0^{\infty}\frac 1{(1+x^2)(1+a^2x^2)}\,dx&=&\frac 1{1-a}\int_0^{\infty}\left(\frac{1}{1+x^2}-\frac{a^2}{1+a^2x^2}\right)\,dx\\ &=&\frac{\pi}{2(1+a)} \end{eqnarray*}$$
となります. ここから, ある定数$C$があって,
$$\begin{eqnarray*} \int_0^{\infty}\frac{\arctan ax}{x(1+x^2)}\,dx=\frac{\pi}{2}\ln(1+a)+C \end{eqnarray*}$$
となりますが, $a=0$として, $C=0$を得ます.
$$\begin{eqnarray*} \int_0^{\infty}\frac{\arctan ax}{x(1+x^2)}\,dx=\frac{\pi}{2}\ln(1+a) \end{eqnarray*}$$
これを用いて, 与えられた積分を考えていきます.

$$\begin{eqnarray*} &&\int_0^{\pi/2}\frac 1{\tan x}\ln\left(\frac{1+(\tan x+\sinh y)^2\cosh^2y}{1+(\tan x-\sinh y)^2\cosh^2 y}\right)\,dx\\ &=&\int_0^{\infty}\frac{1}{x(1+x^2)}\ln\left(\frac{1+(x+\sinh y)^2\cosh^2 y}{1+(x-\sinh y)^2\cosh^2 y}\right)\,dx\\ &=&2\int_0^{\infty}\frac{1}{x(1+x^2)}\ln\left|\frac{1+i((\sinh y+x)\cosh y)}{1+i((\sinh y-x)\cosh y)}\right|\,dx\\ &=&2\Re\int_0^{\infty}\frac 1{x(1+x^2)}\ln\left(\frac{1+i((\sinh y+x)\cosh y)}{1+i((\sinh y-x)\cosh y)}\right)\,dx\\ &=&-4\Re\left( i\int_0^{\infty}\frac 1{x(1+x^2)}\arctan\left(\frac{x\cosh y}{1+i\sinh y\cosh y}\right)\,dx\right)\\ &=&4\Im\left(\frac{\pi}{2}\ln\left(1+\frac{\cosh y}{1+i\sinh y\cosh y}\right)\right)\\ &=&2\pi\arctan\left(\frac{\sinh y\cosh^2 y}{1+\cosh y+\sinh^2 y\cosh^2 y}\right) \end{eqnarray*}$$
となりました.

なんか結果が合いませんが, 数値的には確かに合ってたので, 次の問題にいきましょう.

6個目( https://twitter.com/integralsbot/status/1361144393151909888/photo/1 )

$$\begin{eqnarray*} \int_0^{\pi}\arctan\ln\frac{\sin x}{x}\,dx=-\pi\arctan\frac{2\ln 2}{\pi} \end{eqnarray*}$$

なにこれ, って感じですね. 少し考えても全く分からなかなったので, とりあえず数値を確認してみました.

$$\begin{eqnarray*} \int_0^{\pi}\arctan\ln\frac{\sin x}{x}\,dx&\approx&-1.600756\\ -\pi\arctan\frac{2\ln 2}{\pi}&\approx&-1.305556 \end{eqnarray*}$$
誤差にしては大きいかと思ったけど, 左辺が数値計算に向いてないという可能性もありますね, 僕には解けません.

7個目( https://twitter.com/integralsbot/status/1361150961683623938/photo/1 )

$$\begin{eqnarray*} \int_0^{\infty}\frac{x}{(1+x^2)^2}\frac{dx}{\tanh\frac{\pi x}{2}}=\frac{\pi^2}{8}-\frac 12 \end{eqnarray*}$$

これはディガンマ関数の微分ですね. Binetの第2公式を2階微分すると出るはずなので, やってみます.

Binetの第2公式
$$\begin{eqnarray*} \ln\Gamma(z)=\frac 12\ln 2\pi+\left(z-\frac 12\right)\ln z-z+2\int_0^{\infty}\frac{\arctan\frac{x}{z}}{e^{2\pi x}-1}\,dx \end{eqnarray*}$$
を二階微分すると,
$$\begin{eqnarray*} \psi'(z)=\frac 1z+\frac 1{2z^2}+4z\int_0^{\infty}\frac{x}{(z^2+x^2)^2(e^{2\pi x}-1)}\,dx \end{eqnarray*}$$
よって,
$$\begin{eqnarray*} \int_0^{\infty}\frac{x}{(z^2+x^2)^2(e^{2\pi x}-1)}\,dx=\frac 1{4z}\left(\psi'(z)-\frac 1z-\frac 1{2z^2}\right) \end{eqnarray*}$$
これを用いて,
$$\begin{eqnarray*} \int_0^{\infty}\frac{x}{(1+x^2)^2\tanh\frac{\pi x}{2}}\,dx&=&\int_0^{\infty}\frac{x}{(1+x^2)^2}\,dx+2\int_0^{\infty}\frac{x}{(1+x^2)^2(e^{\pi x}-1)}\,dx\\ &=&\left[-\frac{1}{2(1+x^2)}\right]_0^{\infty}+\frac 12\int_0^{\infty}\frac{x}{\left(\left(\frac 12\right)^2+x^2\right)^2(e^{2\pi x}-1)}\,dx\\ &=&\frac 12+\frac 14\psi'\left(\frac 12\right)-1\\ &=&\frac{\pi^2}{8}-\frac 12 \end{eqnarray*}$$

8個目( https://twitter.com/integralsbot/status/1361151633397338112 )

$f$を周期$T$の奇関数であるとき,
$$\begin{eqnarray*} \int_0^{\infty}\frac{f(x)}{x}\,dx=\frac{\pi}{T}\int_0^{T/2}\frac{f(x)}{\tan\frac{\pi x}{T}}\,dx \end{eqnarray*}$$

こういうのはとりあえず積分区間を無限にしておくといい感じになる気がします.

$\frac{f(x)}{x}$は偶関数だから,
$$\begin{eqnarray*} \int_0^{\infty}\frac{f(x)}{x}\,dx&=&\frac 12\int_{-\infty}^{\infty}\frac{f(x)}{x}\,dx\\ &=&\frac 12\sum_{n\in\mathbb{Z}}\int_{nT}^{(n+1)T}\frac{f(x)}{x}\,dx\\ &=&\frac 12\sum_{n\in\mathbb{Z}}\int_0^T\frac{f(x)}{x+nT}\,dx\\ &=&\frac 14\sum_{n\in\mathbb{Z}}\int_0^Tf(x)\left(\frac{1}{x-nT}+\frac{1}{x+nT}\right)\,dx \end{eqnarray*}$$
ここで, $\pi\cot\pi z$の部分分数展開,
$$\begin{eqnarray*} \pi\cot\pi z&=&\frac 12\sum_{n\in\mathbb{Z}}\left(\frac 1{z-n}+\frac 1{z+n}\right) \end{eqnarray*}$$
を用いると,
$$\begin{eqnarray*} &&\frac 14\sum_{n\in\mathbb{Z}}\int_0^Tf(x)\left(\frac{1}{x-nT}+\frac{1}{x+nT}\right)\,dx\\ &=&\frac {\pi}{2T}\int_0^Tf(x)\cot\frac{\pi x}{T}\,dx\\ &=&\frac{\pi}{2T}\left(\int_0^{T/2}f(x)\cot\frac{\pi x}{T}\,dx+\int_0^{T/2}f(T-x)\cot\frac{\pi(T-x)}{T}\,dx\right)\\ &=&\frac{\pi}{T}\int_0^{T/2}f(x)\cot\frac{\pi x}{T}\,dx \end{eqnarray*}$$
となって, 示すことができました.

9個目( https://twitter.com/integralsbot/status/1361159002441220096 )

$$\begin{eqnarray*} \int_0^1\tan^{-1}\frac{\tanh^{-1} x-\tan^{-1} x}{\pi+\tanh^{-1} x-\tan^{-1} x}\frac{dx}{x}=\frac{\pi}{8}\ln\frac{\pi^2}{8} \end{eqnarray*}$$

見かけ倒しではなく, 全く解ける気がしません. 気を取り直して次いきましょう.

10個目( https://twitter.com/integralsbot/status/1361168004730540032 )

$$\begin{eqnarray*} \int_0^{\pi/4}\frac{\tan^{2e}x-2\sin^2 x}{\sin 2x\ln\tan x}\,dx=\frac 12\left(1+\ln\frac{\pi}2\right) \end{eqnarray*}$$

これは解ける, 見た瞬間そう思いました.

$0\lt a$としておきます. $t=\tan x$と置換して,
$$\begin{eqnarray*} \int_0^{\pi/4}\frac{\tan^a x-2\sin^2x}{\sin 2x\ln\tan x}\,dx&=&\int_0^1\frac{t^a-2\frac{t^2}{1+t^2}}{\frac{2t}{1+t^2}\ln t}\frac{dt}{1+t^2}\\ &=&\frac 12\int_0^1\left(\frac{t^{a-1}}{\ln t}-\frac{2t}{(1+t^2)\ln t}\right)\,dt\\ &=&\frac 12\int_0^1\frac{t^{a-1}-1}{\ln t}\,dt+\frac 12\int_0^1\left(\frac 1{\ln t}-\frac{2t}{(1+t^2)\ln t}\right)\,dt \end{eqnarray*}$$
初項はFrullani integral
$$\begin{eqnarray*} \int_0^{\infty}\frac{f(ax)-f(bx)}{x}\,dx=(f(\infty)-f(0))\ln\frac ab \end{eqnarray*}$$
より,
$$\begin{eqnarray*} \frac 12\int_0^1\frac{t^{a-1}-1}{\ln t}\,dt&=&-\frac 12\int_0^{\infty}\frac{e^{-ax}-e^{-x}}{x}\,dx\\ &=&\frac 12\ln a \end{eqnarray*}$$
第2項は
$$\begin{eqnarray*} \frac 12\int_0^1\left(\frac 1{\ln t}-\frac{2t}{(1+t^2)\ln t}\right)\,dt=-\frac 12\int_0^{\infty}\left(\frac{e^{-x}}{x}-\frac{2e^{-2x}}{x(1+e^{-2x})}\right)\,dx \end{eqnarray*}$$
ここで, Mellin変換,
$$\begin{eqnarray*} &&\int_0^{\infty}x^{s-1}\left(e^{-x}-\frac{2e^{-2x}}{1+e^{-2x}}\right)\,dx\\ &=&\Gamma(s)-2\int_0^{\infty}\frac{x^{s-1}e^{-2x}}{1+e^{-2x}}\,dx\\ &=&\Gamma(s)-2^{1-s}\int_0^{\infty}\frac{x^{s-1}}{e^x+1}\,dx\\ &=&\Gamma(s)(1-2^{1-s}\eta(s)) \end{eqnarray*}$$
において, $s\to 0$とすることで, $\eta(0)=\frac 12, \eta'(0)=\ln\frac{\pi}{2}$を用いて,
$$\begin{eqnarray*} \int_0^{\infty}\left(\frac{e^{-x}}{x}-\frac{2e^{-2x}}{x(1+e^{-2x})}\right)\,dx&=&\lim_{s\to 0}\Gamma(s)(1-2^{1-s}\eta(s))\\ &=&-\lim_{s\to 0}2^{1-s}\eta(s)\left(-\ln 2+\frac{\eta'(s)}{\eta(s)}\right)\\ &=&\ln 2-\ln\frac{\pi}{2} \end{eqnarray*}$$
よって, 求める積分は
$$\begin{eqnarray*} \frac 12\left(\ln a - \ln 2 +\ln\frac{\pi}2\right)&=&\frac 12\left(\ln \frac a2 +\ln\frac{\pi}2\right)\\ \end{eqnarray*}$$
となり, $a=2e$とすればよい.

今回は10問中8問解けましたが, 解けないものには全く手がつけられません, これから, とりあえず順番に取り組んでいきたいと思います.

投稿日:202134

この記事を高評価した人

高評価したユーザはいません

この記事に送られたバッジ

バッジはありません。

投稿者

Wataru
Wataru
395
24735
超幾何関数, 直交関数, 多重ゼータ値などに興味があります

コメント

他の人のコメント

コメントはありません。
読み込み中...
読み込み中